1answer.
Ask question
Login Signup
Ask question
All categories
  • English
  • Mathematics
  • Social Studies
  • Business
  • History
  • Health
  • Geography
  • Biology
  • Physics
  • Chemistry
  • Computers and Technology
  • Arts
  • World Languages
  • Spanish
  • French
  • German
  • Advanced Placement (AP)
  • SAT
  • Medicine
  • Law
  • Engineering
Nadusha1986 [10]
3 years ago
11

Match the equation with its corresponding solution for x . 9-x/5=3

Mathematics
2 answers:
dlinn [17]3 years ago
4 0
Hi there!

To solve for x, we need to simplify:

9 - x/5 = 3
-x/5 = -6
x/5 = 6
x = 30

Hope this helps!
Vladimir [108]3 years ago
3 0

Answer:

x = 30 .

Step-by-step explanation:

Given: 9-\frac{x}{5} = 3

To find: Solution for x .

Solution: We have given that

9-\frac{x}{5} = 3  

On multiplying both side by 5 .

9 *5 -\frac{x* 5}{5} = 3*5  .

45 - x =15

On subtracting both side by 45 .

45 -45 - x =15 - 45

-x = - 30

On multiplying both side by -1 .

x = 30 .

Therefore, x = 30 .

You might be interested in
A local Elementary School has 560 students. A Random Sample of 50
Komok [63]
Well since there was 560 students the it was 560/560. now that 50 got selected the total portion would be 560/6
7 0
2 years ago
James is studying different kinds of weather.
ivann1987 [24]

Answer:

i got b and h{i hope this helps}

Step-by-step explanation:

6 0
3 years ago
How many kilograms are in 44 pounds? 1 kg=2.2lbs
Lena [83]

Answer:

20 kilograms

Step-by-step explanation:

8 0
3 years ago
Read 2 more answers
What is the radius of a sphere with a volume of 1/6 pie
Yakvenalex [24]

Answer:

\large\boxed{the\ radius\ R=\dfrac{1}{2}}

Step-by-step explanation:

The formula of a volume of a sphere:

V=\dfrac{4}{3}\pi R^3

We have

V=\dfrac{1}{6}\pi

Substitute:

\dfrac{4}{3}\pi R^3=\dfrac{1}{6}\pi        <em>divide both sides by π</em>

\dfrac{4}{3}R^3=\dfrac{1}{6}          <em>multiply both sides by 3</em>

3\!\!\!\!\diagup^1\cdot\dfrac{4}{3\!\!\!\!\diagup_1}R^3=3\!\!\!\!\diagup^1\cdot\dfrac{1}{6\!\!\!\!\diagup_2}

4R^3=\dfrac{1}{2}          <em>divide both sides by 4</em>

R^3=\dfrac{1}{2}:4\\\\R^3=\dfrac{1}{2}\cdot\dfrac{1}{4}\\\\R^3=\dfrac{1}{8}\to R=\sqrt[3]{\dfrac{1}{8}}\\\\R=\dfrac{\sqrt1}{\sqrt8}\\\\R=\dfrac{1}{2}

3 0
3 years ago
14. Which number sentence is true if the
valentinak56 [21]

Answer:

D

Step-by-step explanation:

Here, we want to know which of the options would be true if we replace the term in the box ( D) by the value 1,537.

The correct option here is thus D

What we have here would be

1537 > 1,357

This can be read as 1537 is greater than 1,357 which is correct

6 0
3 years ago
Other questions:
  • What is the slope-intercept form in the picture
    7·1 answer
  • How were the numbers 1 10 100 and 1000 written by romans?
    12·1 answer
  • Emma bought presents for her mom and her grandmamma for Mother's Day. She bought 1 box of strawberries with 24 strawberries in i
    7·1 answer
  • Question 36 of 40
    8·1 answer
  • Consider a comparison of two models. The "complete" model has both curvature and interaction. The "reduced" model has curvature,
    6·1 answer
  • MARKING BRAINIEST PLZ HELP ASAP!!!!
    12·1 answer
  • 3/4 of the class are girls. 1/6 of the girls have long hair. What fraction of the class with long hair? Place your answer in sim
    14·1 answer
  • Find the volume of the triangular pyramid to the nearest whole number (number only in your answer) Please help....
    12·2 answers
  • please help me anyone ?!!!!!
    14·1 answer
  • N is an integer.<br> Write the values of n such that -15 &lt; 3 ≤ 6<br><br> Help please ❤️❤️
    10·2 answers
Add answer
Login
Not registered? Fast signup
Signup
Login Signup
Ask question!